Logging now

Algebra Level 2

{ log 100 x + y = 1 2 log 10 y log 10 x = log 100 4 \large{\begin{cases}{\log_{100}|x+y|=\frac{1}{2}} \\ { \log_{10}y - \log_{10}|x|=\log_{100}4}\end{cases}} The solution set of the equations above is?

For more questions on logarithms try this set Logs of logs .
{ 10 , 20 } , { 10 3 , 20 3 } \{-10,-20 \},\left \{\frac{-10}{3},\frac{20}{3} \right \} { 10 , 20 } , { 10 3 , 20 3 } \{10,20\},\left \{\frac{-10}{3},\frac{-20}{3} \right \} { 10 , 20 } , { 10 3 , 20 3 } \{-10,20\}, \left \{\frac{10}{3},\frac{20}{3} \right \} { 10 , 20 } , { 10 3 , 20 3 } \{-10,-20\}, \left \{\frac{-10}{3},\frac{-20}{3} \right \}

This section requires Javascript.
You are seeing this because something didn't load right. We suggest you, (a) try refreshing the page, (b) enabling javascript if it is disabled on your browser and, finally, (c) loading the non-javascript version of this page . We're sorry about the hassle.

2 solutions

The first equation requires that x + y = 10 , |x + y| = 10, which is represented by the lines x + y = 10 y = x + 10 x + y = 10 \Longrightarrow y = -x + 10 and x + y = 10 y = x 10. x + y = -10 \Longrightarrow y = -x - 10.

The second equation requires that

log 10 ( y x ) = log 10 ( 4 ) log 10 ( 100 ) = log 10 ( 2 ) y = 2 x . \log_{10}\left(\dfrac{y}{|x|}\right) = \dfrac{\log_{10}(4)}{\log_{10}(100)} = \log_{10}(2) \Longrightarrow y = 2|x|.

In the first quadrant the solution will be where y = x + 10 y = -x + 10 intersects y = 2 x , y = 2x, , i.e.,

2 x = x + 10 x = 10 3 y = 20 3 . 2x = -x + 10 \Longrightarrow x = \dfrac{10}{3} \Longrightarrow y = \dfrac{20}{3}.

In the second quadrant we require that

x + 10 = 2 x x = 10 y = 20. -x + 10 = -2x \Longrightarrow x = -10 \Longrightarrow y = 20.

Thus the solution set is ( 10 , 20 ) , ( 10 3 , 20 3 ) . \boxed{(-10,20), \left(\dfrac{10}{3}, \dfrac{20}{3}\right)}.

JEE style : for 1st condition to be true , x + y |x+y| should be 10 which is given only in the one of the given options. So that's the correct answer provided that the question is framed with correct answer. :P

Sandeep Bhardwaj - 6 years, 2 months ago
Aman Gautam
Apr 5, 2015

a lazy approach...!
as y cannot be negative ,eliminate the other options which have y as negative..!!!

Moderator note:

Please refrain from posting an antisolution as an official solution.

0 pending reports

×

Problem Loading...

Note Loading...

Set Loading...